11. To wrap gift boxes, Joelle uses 24 yards of ribbon, which is 86 of her total amount of ribbon. How many yards of ribbon does she have in all? A 32 B 192 C 300 D 1,920​

Answers

Answer 1

Joelle has a total of 32 yards of ribbon in all, the correct option is A.

Let's use x to represent Joelle's total amount of ribbon. We know that 24 yards of the ribbon represent 86% of her total amount of ribbon, which can be expressed as:

24 = 0.86x

We can solve for x by dividing both sides by 0.86:

x = 27.91

The response options are all integers, thus we must round to the closest whole number because of this. Since 0.91 is greater than or equal to 0.5, we round up to 28.

Therefore, Joelle has a total of

x = 27.91 / 0.86

= 32.44 yards of ribbon.

Once again, we round to the nearest whole number, which is 32.

To learn more about yards follow the link:

https://brainly.com/question/28365360

#SPJ1

The complete question is:

To wrap gift boxes, Joelle uses 24 yards of ribbon, which is 86 of her total amount of ribbon. How many yards of ribbon does she have in all?

A 32

B 192

C 300

D 1920​


Related Questions

in 2005 the population of a district was 35,700 with a continuous annual growth rate of approximately 4%, what will the population be in 2030 according to the exponential growth function?

Answers

The population of a district in 2005 was 35,700 with a continuous annual growth rate of approximately 4%. the population in 2030 will be approximately 97,209 according to the exponential growth function.

The formula for the continuous exponential growth is given by the formula:

P = Pe^(rt)

where,P is the population in the future.

P0 is the initial population.

t is the time.

r is the continuous interest rate expressed as a decimal.

e is a constant equal to approximately 2.71828.In this problem, the initial population P0 is 35,700. The rate r is 4% or 0.04 expressed as a decimal. We want to find the population in 2030, which is 25 years after 2005.

Therefore, t = 25.We will now use the formula:

P = Pe^(rt)P = 35,700e^(0.04 × 25)P = 35,700e^(1)P = 35,700 × 2.71828P = 97,209.09.

for such more question on population

https://brainly.com/question/25630111

#SPJ11

Answer: I got 97,042.7

Step-by-step explanation:

Duri wants their backpack to weigh less than 45 pounds.
Use w to represent weights where Duri can carry their backpack.

Answers

This inequality represents that the weight of Duri's backpack is w < 45

How to represent the backpack as an expression

Given that

Weight = Less than 45 pounds

We can use the inequality symbol to represent Duri's weight limit as follows:

w < 45

This inequality states that the weight of Duri's backpack, represented by w, must be less than 45 pounds.

Any weight value for w that satisfies this inequality is within Duri's weight limit and can be carried in their backpack.

For example, if Duri's backpack weighs 30 pounds, then w = 30 satisfies the inequality w < 45, so Duri can carry this weight.

However, if the backpack weighs 50 pounds, then w = 50 does not satisfy the inequality w < 45, so Duri cannot carry this weight.

Read more about inequality at

https://brainly.com/question/25275758

#SPJ1

Unit 1 is m feet, unit 2 is n feet unit 3 is k dollars per square feet, write an expression that could result in a final unit measure of dollars

Answers

The expression that could result in a final unit measure of dollars is given by (mnk²) dollars.

To write an expression that could result in a final unit measure of dollars, we need to consider the units given.

We have three units, i.e., m feet, n feet, and k dollars per square foot.

To convert these units to dollars, we need to multiply the given units with appropriate conversion factors.

The conversion factor for the given units is as follows:

Unit 1: 1 foot = k dollars (Conversion factor)

Unit 2: 1 foot = k dollars (Conversion factor)

Unit 3: 1 square foot = 1 x k dollars = k dollars (Conversion factor)

Now, let us write the expression that results in a final unit measure of dollars.

The expression should be written in HTML format.

Let us begin: Final Unit Measure = (Unit 1) x (Unit 2) x (Unit 3)

Final Unit Measure = (m feet) x (n feet) x (k dollars per square foot)

Final Unit Measure = m x k dollars x n x k dollars per square foot

Final Unit Measure = (m x k x n x k) dollars

Final Unit Measure = (mnk²) dollars.

Thus, the expression that could result in a final unit measure of dollars is given by (mnk²) dollars.

For similar question on final unit measure.

https://brainly.com/question/18828615

#SPJ11

Find the x-intercept of 3 tan(3x) over the interval (pi/6,3pi/6)
Express your answer in terms of pi.

Answers

The x-intercepts of the function 3 tan(3x) over the interval (π/6, 3π/6) are:

x = π/3 and x = 2π/3

What is function ?

A function is a mathematical object that takes one or more inputs, called the arguments or variables, and produces a unique output. The output is determined by a set of rules that specify how the function operates on the inputs. In other words, a function is a relationship between inputs and outputs.

Functions are typically denoted by a symbol or a name, such as f(x) or g(t). The input is usually represented by a variable, such as x or t, while the output is represented by the function value, such as f(x) or g(t).

Functions are used extensively in mathematics, science, engineering, and many other fields. They provide a way to model and analyze real-world phenomena, and they are essential tools for solving many problems in these fields. Examples of functions include polynomial functions, exponential functions, trigonometric functions, and logarithmic functions.

To find the x-intercept of the function 3 tan(3x) over the given interval, we need to find the values of x where the function equals zero.

Let's first simplify the function:

3 tan(3x) = 0

tan(3x) = 0

We know that tan(π/2) is undefined and that tan(π) = 0. Since the period of the tangent function is π, we can say that:

tan(3x) = 0 --> 3x = nπ for n ∈ ℤ

Now we solve for x:

3x = nπ

x = nπ/3

Since the interval is (π/6, 3π/6), we need to find the values of x that satisfy:

π/6 < x < 3π/6

π/6 < nπ/3 < 3π/6

1/2 < n < 3/2

So the values of x that satisfy the given condition are:

x = π/3 and x = 2π/3

Therefore, the x-intercepts of the function 3 tan(3x) over the interval (π/6, 3π/6) are:

x = π/3 and x = 2π/3

Expressed in terms of π, the x-intercepts are:

π/3π and 2π/3π, which simplify to:

x = 1/3 and x = 2/3.

To know more about Trigonometry visit :-

https://brainly.com/question/13729598

#SPJ1

9 N = 480 ´ 109
(a) Write N as a number in standard form.
(1)
(b) Write N as a product of powers of its prime factors.
Show your working clearly.
(3)
(c) Find the largest factor of N that is an odd number.

Answers

Answer:

(a) To write 9 N as a number in standard form, we need to express it as a number between 1 and 10 multiplied by a power of 10. To do this, we can divide 9 N by 10 until we get a number between 1 and 10:

9 N = 480 × 10^9

9 N ÷ 10 = 48 × 10^9

9 N ÷ 10^2 = 4.8 × 10^9

9 N ÷ 10^3 = 0.48 × 10^9

9 N ÷ 10^4 = 0.048 × 10^9

9 N ÷ 10^5 = 0.0048 × 10^9

Therefore, 9 N = 4.8 × 10^10.

(b) To write N as a product of powers of its prime factors, we can first factorize N:

480 × 10^9 = 2^5 × 3 × 5 × 10^9

Then, we can express 10^9 as 2^9 × 5^9 and substitute it in the factorization:

2^5 × 3 × 5 × 2^9 × 5^9 = 2^14 × 3 × 5^10

Therefore, N = 2^14 × 3 × 5^10.

(c) To find the largest factor of N that is an odd number, we need to remove all factors of 2 from the factorization of N. We can do this by dividing N by 2 as many times as possible:

N = 2^14 × 3 × 5^10

N ÷ 2 = 2^13 × 3 × 5^10

N ÷ 2^2 = 2^12 × 3 × 5^10

N ÷ 2^3 = 2^11 × 3 × 5^10

N ÷ 2^4 = 2^10 × 3 × 5^10

N ÷ 2^5 = 2^9 × 3 × 5^10

N ÷ 2^6 = 2^8 × 3 × 5^10

N ÷ 2^7 = 2^7 × 3 × 5^10

N ÷ 2^8 = 2^6 × 3 × 5^10

N ÷ 2^9 = 2^5 × 3 × 5^10

N ÷ 2^10 = 2^4 × 3 × 5^10

N ÷ 2^11 = 2^3 × 3 × 5^10

N ÷ 2^12 = 2^2 × 3 × 5^10

N ÷ 2^13 = 2 × 3 × 5^10

N ÷ 2^14 = 3 × 5^10

Therefore, the largest factor of N that is an odd number is 3 × 5^10.

PLEASEEEEE HELPPPPPPP!!!!!!!

A line segment contains endpoints A(-1, 2) and B(2, 5).
Determine the point that partitions line segment AB into a 3: 6 ratio.

A 4,5/3
B 0,3
C 1/3,3
D -2,1

Answers

Answer:

We can find the point that partitions line segment AB into a 3:6 ratio by using the formula for finding a point that divides a line segment into two parts in a given ratio.

Let's call the point we're looking for "P". According to the formula, the coordinates of point P can be found using the following equations:

x-coordinate of P = [(6 * x-coordinate of A) + (3 * x-coordinate of B)] / 9

y-coordinate of P = [(6 * y-coordinate of A) + (3 * y-coordinate of B)] / 9

Using the coordinates of points A and B given in the problem, we can plug them into these equations and simplify to find the coordinates of point P:

x-coordinate of P = [(6 * -1) + (3 * 2)] / 9 = 0

y-coordinate of P = [(6 * 2) + (3 * 5)] / 9 = 3.33 (rounded to two decimal places)

Therefore, the point that partitions line segment AB into a 3:6 ratio is approximately (0, 3.33), which is closest to option A: 4,5/3.

solve the system 2 -1 5 -2 with -2 -1 . give your solution in real form. , . an ellipse with counterclockwise orientation 1. describe the trajectory.

Answers

The solution of the system 2 -1 5 with -2 -1 in real form is (x, y) = (7/4, 7/2).

To solve the system with the given coefficients, first, let's rewrite it as a linear system of equations:

2x - y = 5
-2x - y = -2

Now, let's solve this system step-by-step:

Step 1: Solve for y in the first equation:
y = 2x - 5

Step 2: Substitute the expression for y from Step 1 into the second equation:
-2x - (2x - 5) = -2

Step 3: Simplify and solve for x:
-2x - 2x + 5 = -2
-4x = -7
x = 7/4

Step 4: Substitute the value of x back into the expression for y:
y = 2(7/4) - 5
y = 7/2

The solution in real form is (x, y) = (7/4, 7/2).

Regarding the ellipse with counterclockwise orientation, the trajectory can be described as a smooth, closed curve in the shape of an ellipse. The ellipse will have a major and minor axis, with the points on the ellipse moving continuously in a counterclockwise direction along the curve.

To learn more about trajectory: https://brainly.com/question/13244761

#SPJ11

Compare the amount of sand in the top cone of the hourglass to the amount there will be when the height of the sand in the top cone is only 1 inch.
HINT: The cones are similar

Answers

the amount of sand in the top cone when the height of the sand is only 1 inch is (h-1)/h times the amount of sand in the top cone originally.

the cones are similar, their volumes are proportional to the cube of their heights. Let's denote the height of the top cone as h, and the radius of the top and bottom bases as r. Then, the volume of the top cone can be expressed as:

V₁ = (1/3)π[tex]r^2[/tex]h

If the height of the sand in the top cone is reduced to 1 inch, then the height of the remaining sand in the top cone is (h-1) inches. The volume of the remaining sand in the top cone can be expressed as:

V₂ = (1/3)π[tex]r^2[/tex](h-1)

To compare the amount of sand in the top cone in these two scenarios, we can take the ratio of their volumes:

V₂/V₁ = [(1/3)π[tex]r^2[/tex](h-1)] / [(1/3)π[tex]r^2[/tex]h] = (h-1)/h

Learn more about proportional here:

https://brainly.com/question/28979975

#SPJ1

< Back to task
In the quadrilateral below, angles DAB and BCD are the same size.
What is the size of angle DAB?

D
228

34° -B
Answer >

Answers

The size of angle DAB in the quadrilateral is 49°.

How to find the size of angle DAB?

The sum of the interior angles of a quadrilateral is 360°. We can say:

∠A + ∠B + ∠C + ∠D = 360°

∠A + 34° + ∠C + 228° = 360°

∠A  + ∠C + 262° = 360°

∠A  + ∠C = 360 - 262

∠A  + ∠C = 98

Since angles DAB and BCD are the same size. This implies ∠A  = ∠C. Thus:

∠A  + ∠A = 98

2∠A  = 98

∠A = 98/2

∠A  = 49°

Therefore, the size of angle DAB is 49°.

Learn more about quadrilateral on:

https://brainly.com/question/23935806

#SPJ1

Complete Question

Check the attached image

You are offered a job that pays ​$34,000 during the first​ year, with an annual increase of ​6% per year beginning in the second year. That​ is, beginning in year​ 2, your salary will be 1.06 times what it was in the previous year. What can you expect to earn in your fourth year on the​ job? Round your answer to the nearest dollar.

Answers

Year 1 salary: $34,000

Year 2 salary: 1.06 x $34,000 = $36,040

Year 3 salary: 1.06 x $36,040 = $38,316.40

Year 4 salary: 1.06 x $38,316.40 = $40,850.38

Rounding to the nearest dollar, you can expect to earn $40,850 in your fourth year on the job.

Could someone please help me I don’t understand this

Answers

Slope = rise / run



Solution:


DC= 1 / 3


• The slope of DC is the same as AB

9. The linear regression equation is = 34.38x - 91.75. Use the equation to predict how far this
4.38x-91-75 Use
person will travel after 10 hours of driving.

Answers

The answer of the given question based on the  linear regression  is , the predicted distance the person will travel after 10 hours of driving is approximately 252.05 miles.

What is Distance?

Distance is measurement of length between the two points or objects. It is a scalar quantity that only has a magnitude and no direction. In mathematics, distance can be measured in various units such as meters, kilometers, miles, or feet, depending on the context.

Distance can be calculated using the distance formula, which is based on the Pythagorean theorem in two or three dimensions.

Assuming the equation you meant to write is y = 34.38x - 91.75, where y is the predicted distance traveled in miles and x is the number of hours driven, we can use this equation to predict how far the person will travel after 10 hours of driving:

y = 34.38x - 91.75

y = 34.38(10) - 91.75

y = 343.8 - 91.75

y = 252.05

Therefore, the predicted distance the person will travel after 10 hours of driving is approximately 252.05 miles.

To know more about Equation visit:

https://brainly.com/question/9312365

#SPJ9

During 10 hours of driving, the projected distance according to linear regression is roughly 252.05 miles.

What is Distance?

The term "distance" refers to the length between two points or objects. Having merely a magnitude and no direction, it is a scalar quantity. Depending on the situation, distance in mathematics can be expressed in a variety of ways, including meters, kilometers, miles, or feet.

The distance formula, which depends on the Pythagorean theorem in either two or three dimensions, can be used to compute distance.We may use this equation to forecast how far the individual would go after 10 hours of driving, assuming the equation you meant to write is

y = 34.38x - 91.75, where y is the expected distance travelled in miles and x is the number of hours driven:

y = 34.38x - 91.75

y = 34.38(10) - 91.75

y = 343.8 - 91.75

y = 252.05

The estimated distance that the driver will cover after 10 hours on the road is 252.05 miles.

To know more about linear regression, visit:

https://brainly.com/question/30063703

#SPJ9

The complete question is,

The equation for linear regression is = 34.38x - 91.75. Calculate this person's estimated distance after 10 hours of driving using the equation: 4.38x-91-75.

Complete the table
Original Price
Percent of Discount
15%
Sale Price
$146.54

Answers

The final result for the money is [tex]172.40[/tex] after you multiply it by a percent.

What does a maths percent mean?

In essence, percentages are fractions with a 100 as the denominator. We place the percent sign (%) next to the number to indicate that the number is a percentage.

What does the word "percentage" actually mean?

Rather than being stated as a fraction, a percent is a piece of an entire thing expressed as just a number between zero and 100. Nothing is zero percent; everything is 100 percent; half of something is 50 percent; and nothing is zero percent. You divide the part of the total by its entirety and multiply the result by 100 to get the percentage.

[tex]w-[0.15]=146.54[/tex]

so [tex]w-85p=146.54[/tex]

[tex]146.54[/tex] divided by [tex]80=1.724[/tex] so when you make it a percent it turns it into [tex]172.40[/tex] which is the final answer for the money.

To know more about percent visit:

https://brainly.com/question/3227167

#SPJ1

PLEASE HELP - WORTH 30 POINTS

Answers

Answer: 1,023 combinations & 4/5 chance

Step-by-step explanation: A bunch of weird math with the first answer, but the 2nd answer is just every clothing article that isn't red over the total amount of articles simplified (8/10 = 4/5).

Hope this helped!

If h = 13 units and r = 6 units, then what is the volume of the cone shown above?

Answers

The vοlume οf the cοne is 156π cubic units

What is cοne?

A cοne is a three-dimensiοnal geοmetric fοrm with a flat base and a smοοth tapering tip οr end. A cοne is made up οf a cοllectiοn οf line segments, half-lines, οr lines that link the apex—the cοmmοn pοint—tο every pοint οn a base that is in a plane οther than the apex.

Given,

h = 13 units and r = 6 units

We knοw the fοrmula tο determine vοlume οf cοne; that is

(1/3)πr²h

Where r= radius οf the cοne

h=  height οf the cοne

We put the value οf h and r in the fοrmula

(1/3)πr²h

= (1/3)π6² *13

=156π cubic units

The vοlume οf the cοne is 156π cubic units

Hence the cοrrect answer is 156π cubic units

To learn more about cone from the given link

https://brainly.com/question/1082469

#SPJ9

you work as a cashier in a supermarket. on saturday, you had 135 customers. on sunday, you had 90 customers. what is the approximate percent decrease

Answers

The approximate percent decrease in the number of customers from Saturday to Sunday is 33%. Therefore, the answer is option A: 33%

To find the approximate percent decrease in the number of customers from Saturday to Sunday, we first need to calculate the decrease in the number of customers:

Decrease = Saturday customers - Sunday customers

Decrease = 135 - 90

Decrease = 45

The percent decrease can be found by taking the decrease as a percentage of the original value (Saturday customers):

Percent decrease = (Decrease / Saturday customers) x 100%

Percent decrease = (45 / 135) x 100%

Percent decrease = 33.33%

Rounding to the nearest percent, we get that the approximate percent decrease in the number of customers from Saturday to Sunday is 33%. Therefore, the answer is option A: 33%.

Learn more about  calculations of percent decrease:https://brainly.com/question/30404835

#SPJ11

Your question is incomplete, but probably the complete question is :

You work as a cashier in a supermarket. On Saturday, you had 135 customers. On Sunday, you had 90 customers. What is the approximate percent decrease in the number of customers from Saturday to Sunday?

33%

45%

50%

90%

225%

By the 46th day, there are 400 water lilies in the pond. the estimate you made:____.a. close to 46 (or exactly right) because it was estimated well.b. too low because the quick exponential growth was not accounted for. c. too high because the exponential growth was overestimated.

Answers

By the 46th day, there are 400 water lilies in the pond. the estimate you made: a). close to 46 (or exactly right) because it was estimated well.

We know that the regression equation is: y = 3.915(1.106)ˣ and that the pond can hold 400 water lilies.

y = 3.915(1.106)ˣ

Here, y is equal to the number of water lilies the pond can hold.

So, y = 400

And,  x is the required time taken.

So,

400 = 3.915(1.106)ˣ

⇒ (1.106)ˣ = 400/3.915    

⇒ (1.106)ˣ = 102.17

Taking logarithm on both sides, we get

log (1.106)ˣ = log (102.17)

⇒ x log (1.106) = log (102.17) .............. [log aˣ = x loga]

⇒ x = [log (102.17)/log (1.106)]    

⇒ x = 45.92

x ≅ 46

So, the pond will take 46 days to become full.

To learn more about regression equation, click here:

brainly.com/question/30738733

#SPJ4

Complete question:

Regression equation: y = 3.915(1.106)ˣ . The pond can hold 400 water lilies. by what day will the pond be full? write and solve an equation. the pond will be full by the end of day.

Answer:

b

Step-by-step explanation:

just did it

a newsletter publisher believes that under 69% 69 % of their readers own a rolls royce. is there sufficient evidence at the 0.10 0.10 level to substantiate the publisher's claim? state the null and alternative hypotheses for the above scenario.

Answers

No, there is not sufficient evidence to substantiate the claim at the 0.10 level. Null Hypothesis (H0): p ≤ 0.69; Alternative Hypothesis (H1): p > 0.69

A newsletter publisher believes that under 69% of their readers own a Rolls Royce.

No, there is not sufficient evidence to substantiate the claim at the 0.10 level. In order to substantiate the claim at the 0.10 level, the publisher would need to collect data from their readers and perform a hypothesis test to compare the observed percentage of readers with the claimed percentage.

Null Hypothesis: The proportion of the newsletter readers that own a Rolls Royce is less than or equal to 69%.

Null Hypothesis (H0): p ≤ 0.69

Alternative Hypothesis: The proportion of the newsletter readers that own a Rolls Royce is greater than 69%.

Alternative Hypothesis (H1): p > 0.69

To learn more about Null and Alternative Hypothesis link is here

brainly.com/question/30899146

#SPJ4

plot four different points whose -coordinates are half their -coordinates. do these points lie on a line?

Answers

The four points with y-coordinates half their x-coordinates are (0,0), (2,1), (4,2), and (6,3). These points do lie on a line, as they all satisfy the linear equation y = x/2.

To plot four points whose y-coordinates are half their x-coordinates, we can choose any four values of x and then compute the corresponding values of y using the equation y = x/2. For example

If x = 0, then y = 0/2 = 0, so the first point is (0,0).

If x = 2, then y = 2/2 = 1, so the second point is (2,1).

If x = 4, then y = 4/2 = 2, so the third point is (4,2).

If x = 6, then y = 6/2 = 3, so the fourth point is (6,3).

We can plot these points on a coordinate plane

As we can see from the plot, the four points do lie on a straight line. This is because the equation y = x/2 is the equation of a linear function with slope 1/2 and y-intercept 0. Therefore, any two points on this line will have a constant slope between them, and thus the four points will be collinear.

Learn more about coordinates plane here

brainly.com/question/24134413

#SPJ4

What would be the new coordinates of W' after a dilation of 3? W

Answers

The new coordinates would be

W' (12 , 6)

X'( 24 , 18 )

Z'(24 ,6 )

What exactly does coordinate geometry mean?

The term "coordinate geometry" refers to the study of geometry using coordinate points (or analytic geometry). Calculating distances between points, segmenting lines into m:n pieces, finding a line's midpoint, figuring out a triangle's area in the Cartesian plane, and other operations are all achievable with coordinate geometry.

Remember that the rule for a dilation by a factor of k about the origin is

 (x,y) = (kx, ky)

Identify the coordinates of the points W, X and Z. Then, apply a dilation by a factor of 3 about the origin to find W', X' and Z', the new coordinates after the dilation.

  w = (4,2)

   x = ( 8, 6 )

   z = ( 8,2)

Apply a dilation by a factor of 3:

  W(4,2) ⇒ W'(3 * 4, 3 * 2) = W' (12 , 6)

 X(8 ,6 ) ⇒ X'(3 * 8 , 3 * 6 ) = X' ( 24 , 18 )

 Z(8 , 2 ) ⇒ Z'(3*8 , 3 * 2) = Z'(24 ,6 )

Therefore, the new coordinates would be

W' (12 , 6)

X'( 24 , 18 )

Z'(24 ,6 )

Learn more about Coordinate geometry

brainly.com/question/18269861

#SPJ1

The polynomial has been factored completely. What are the zeros of the function?
x2-2x-48-(x+6)(x-8)
x=6 and x=8
x=-6 and x=8
x = 6 and x = -8
x = -6 and x = -8

Answers

The zeroes of the factored polynomial as required to be determined in the task content are: x = -6 and x = 8.

What are the zeroes of the factorised polynomial?

It follows from the task content that the zeroes of the completely factorised polynomial are to be determined.

Since the given polynomial is; x² - 2x - 48 which has been factorised completely to; (x + 6) (x - 8).

Ultimately, the zeroes of the Polynomial are as follows;

x + 6 = 0; x = -6.

and

x - 8 = 0; x = 8.

Conclusively, the zeroes of the polynomial are; x = -6 and x = 8.

Read more on zeroes of a polynomial;

https://brainly.com/question/29415775

#SPJ1

9r subtract three fifths greater than 3 and 9 tenths

Answers

    the baker needs 15 gallons of milk to make 80 chocolate pies for the community festival. To translate the phrase "9r subtract three fifths greater than 3 and 9 tenths" into an expression, we first need to understand what it's asking us to do.  

  "Three fifths greater than 3 and 9 tenths" means we need to add 3 and 9 tenths to three fifths of 3. Three fifths of 3 is 1.8 (since 3/5 * 3 = 9/5 = 1.8), so we can write:

3 + 9/10 + 1.8

   

We can simplify this to a single mixed number by adding the whole numbers and the fractions separately:

3 + 1 + 8/10 + 8/5

= 4 + 1 3/5

= 5 3/5

So "three fifths greater than 3 and 9 tenths" is equal to 5 3/5.

Now we can subtract this value from 9r:

9r - 5 3/5

We can simplify this expression further by converting 5 3/5 to a fraction with a common denominator of 5:

9r - 5 3/5 = 9r - (28/5) = (45/5)r - (28/5) = (9r - 28) / 5

So the final expression is:

(9r - 28) / 5

In summary, "9r subtract three fifths greater than 3 and 9 tenths" can be translated to the expression (9r - 28) / 5. This expression represents a quantity that is 9 times "r" minus 5 3/5. We can simplify this expression further by converting the mixed number to an improper fraction and combining the terms, as shown above.

To know more about common denominator click here:

brainly.com/question/29775115

#SPJ4

What is 4.8 x 0.1 ?


Question :

4.8 x 0.1 =

Answers

Answer: 0.48

Step-by-step explanation:

× 0.1 = ÷10

÷ 0.1 = ×10

× 0.01 = ÷100

÷ 0.01 = ×100

So, we do=

4.8 x 0.1 = 4.8 ÷ 10

= 0.48

So our answer is 0.48

First break up the question.
4.0x0.1=0.4
0.8x0.1=0.08
Then add the answers
0.4+0.08=0.48
The answer is 0.48

after a (not very successful) trick or treating round, candice has 12 tootsie rolls and 10 twizzlers in her pillow case. her mother asks her to share the loot with her three younger brothers. (a) how many different ways can she do this?

Answers

Using the stars and bars technique, Candice can distribute her 24 pieces of candy among her four siblings in 2,925 different ways. If she must give each sibling at least one of each type of candy, there are 67,200 ways to distribute the candy among the four siblings.

(A) To solve this problem, we can use the technique of stars and bars. We have a total of 24 pieces of candy to share among four children. We can represent this using 24 stars, with 3 bars to separate the stars into four groups, one for each child. For example, the following arrangement represents giving 6 pieces of candy to the first child, 10 pieces to the second child, 3 pieces to the third child, and 5 pieces to the fourth child:

*****|**********|***|****

The number of ways to arrange the stars and bars is equal to the number of ways to choose 3 positions out of the 27 possible positions for the stars and bars. Therefore, the number of different ways that Candice can share her candy with her three younger brothers is:

C(27, 3) = 27! / (3! * 24!) = 2925

(B) Now, we need to ensure that each child receives at least one Tootsie roll and one Twizzler. We can give each child one of each candy to start, and then distribute the remaining 13 Tootsie rolls and 7 Twizzlers using the stars and bars technique. We have 13 Tootsie rolls and 7 Twizzlers to distribute among four children, which can be represented using 13 stars and 3 bars for the Tootsie rolls, and 7 stars and 3 bars for the Twizzlers. The number of ways to arrange the stars and bars for each type of candy is:

C(16, 3) = 560 for the Tootsie rolls

C(10, 3) = 120 for the Twizzlers

To find the total number of ways to distribute the candy, we can multiply the number of ways for each type of candy:

560 * 120 = 67200

Therefore, there are 67,200 different ways for Candice to share her candy with her three younger brothers after her mother asks her to give at least one of each type of candies to each of her brothers.

Learn more about combinatorics here: brainly.com/question/13261685

#SPJ4

Complete question:

After a (not very successful) trick or treating round, Candice has 15 Tootsie rolls and 9 Twizzlers in her pillow case. Her mother asks her to share some of the loot with her three younger brothers.

(A) How many different ways can she do this?

(B) How many different ways can she do this after her Mother asks her to give at least one of each type of candies to each of her brothers?

​Write an equation for line t. Show or explain how you determined your equation.
​Enter your equation and your work or explanation in the box provided.

Answers

Answer:

[tex]y - 3 = \frac{2}{3} (x - 3)[/tex]

[tex]y = \frac{2}{3} x + 1[/tex]

[tex]2x - 3y = - 3[/tex]

Step-by-step explanation:

[tex]m = \frac{ - 5 - 3}{ - 9 - 3} = \frac{ - 8}{ - 12} = \frac{2}{3} [/tex]

[tex]y - 3 = \frac{2}{3} (x - 3)[/tex]

[tex]y - 3 = \frac{2}{3} x - 2[/tex]

[tex]y = \frac{2}{3} x + 1[/tex]

[tex]3y = 2x + 3[/tex]

[tex] - 2x + 3y = 3[/tex]

[tex]2x - 3y = - 3[/tex]

DUE FRIDAY WELL WRITTEN ANSWERS ONLY!!!!!!!!!!!
Complete the table

Answers

All the trigonometric values for sin θ, cos θ and tan θ are valued below. Each trigonometric value is mentioned.

sin θ has boundaries from 0 to 1.

sin [tex]-\pi /2[/tex] = -1

sin  [tex]-\pi /3[/tex] = -0.87

sin [tex]-\pi /6[/tex] = -0.5

sin 0 = 0

sin [tex]\pi /6 \\[/tex] = 0.5

sin [tex]\pi /3[/tex] = 0.87

sin [tex]\pi /2[/tex] = 1

sin [tex]2\pi /3[/tex] = [tex]\sqrt{3}/2[/tex]

sin [tex]5\pi /6[/tex] = 1/2

sin [tex]\pi[/tex] = 1

sin [tex]7\pi /6[/tex] = -0.5

sin [tex]4\pi /3[/tex] = -0.87

sin [tex]3\pi /2[/tex] = -1

sin  [tex]5\pi /3[/tex] = -0.87

sin [tex]11\pi /6[/tex] = -0.5

sin [tex]2\pi[/tex] = 0

Similarly cos θ has boundaries.

cos [tex]-\pi /2[/tex] = 0

cos  [tex]-\pi /3[/tex] = 0.5

cos [tex]-\pi /6[/tex] = 0.87

cos 0 = 1

cos [tex]\pi /6 \\[/tex] = 0.87

cos [tex]\pi /3[/tex] = 0.5

cos [tex]\pi /2[/tex] = 0

cos [tex]2\pi /3[/tex] = -0.5

cos [tex]5\pi /6[/tex] = -0.87

cos [tex]\pi[/tex] = -1

cos [tex]7\pi /6[/tex] = -0.87

cos [tex]4\pi /3[/tex] = -0.5

cos [tex]3\pi /2[/tex] = 0

cos  [tex]5\pi /3[/tex] = 0.5

cos [tex]11\pi /6[/tex] = 0.87

cos [tex]2\pi[/tex] = 1

But tan θ has no boundaries.

tan [tex]-\pi /2[/tex] = undefined

tan[tex]-\pi /3[/tex] = -0.8

tan [tex]-\pi /6[/tex] = -1.73

tan 0 = 0

tan[tex]\pi /6 \\[/tex] = [tex]\frac{1}{\sqrt{3} }[/tex]

tan [tex]\pi /3[/tex] = [tex]\sqrt{3}[/tex]

tan [tex]\pi /2[/tex] = undefined

tan [tex]2\pi /3[/tex] = -3

tan [tex]5\pi /6[/tex] = -0.5774

tan [tex]\pi[/tex] = undefined

tan [tex]7\pi /6[/tex] = -1.73

tan[tex]4\pi /3[/tex] = 1.73

tan [tex]3\pi /2[/tex] = undefined

tan [tex]5\pi /3[/tex] = -1.73

tan [tex]11\pi /6[/tex] = -0.58

tan [tex]2\pi[/tex] = 0

Hence, all the values mentioned in the table, were written above.

Learn more about Trigonometric values:

https://brainly.com/question/29069676

#SPJ1

A plane is 148 mi north and 167 mi east of an airport. Find x, the angle the pilot should turn in order to fly directly to the airport. Round your answer to the nearest tenth of a degree​

Answers

Therefore, the pilot should turn by approximately 41.8 degrees to fly directly to the airport.

What is trigonometry?

Trigonometry is a branch of mathematics that deals with the study of the relationships between the sides and angles of triangles. It has applications in various fields, such as engineering, physics, architecture, and astronomy. Trigonometry is based on the use of six fundamental trigonometric functions, which are sine, cosine, tangent, cosecant, secant, and cotangent. These functions are defined in terms of the ratios of the sides of a right triangle. In a right triangle, one angle is a right angle, which measures 90 degrees, and the other two angles are acute angles, which are less than 90 degrees. The three sides of a right triangle are called the hypotenuse, the adjacent side, and the opposite side. The hypotenuse is the longest side, and it is always opposite to the right angle. The adjacent side is the side that is adjacent to the angle of interest, and the opposite side is the side that is opposite to the angle of interest.

Here,

We can use trigonometry to find the angle x that the pilot should turn in order to fly directly to the airport.

First, let's draw a diagram of the situation:

A(airport)

|\

| \

|  \

|   \

|    \

|     \

|      \

|       \

|        \

|         \

|          \

P         x mi

In the diagram, P represents the position of the plane, which is 148 miles north and 167 miles east of the airport A. The line labeled "x mi" represents the distance that the plane needs to fly in order to reach the airport, and the angle x is the angle between the line x mi and the line representing the eastward direction.

To find x, we can use the trigonometric ratio for tangent (tan):

tan(x) = opposite/adjacent

In this case, the opposite side is 148 miles (the distance north of the airport) and the adjacent side is 167 miles (the distance east of the airport). Therefore:

tan(x) = 148/167

Using a calculator, we can find that:

tan(x) ≈ 0.8868

To find x, we need to take the arctangent (tan⁻¹) of both sides:

x = tan⁻¹(0.8868)

Using a calculator, we find that:

x ≈ 41.8°

To know more about trigonometry,

https://brainly.com/question/26719838

#SPJ1

HHHHHHHHHHHHHHHHHHHHHHHHHHHHHHHHHHelp

Answers

Answer:

first box (pies): 3, 6, 9, 12, 15, 18, 33

-> increments of 3

2nd box [cost ($)]: 11, 22, 33, 44, 55, 66, 121

-> increment of 11

Step-by-step explanation:

$44 ÷ 12 pies = $3.67 per 1 pie

3 × $3.67 = $11.01

same process: (number of pies) × $3.67 ≈ COST

Answer:

Step-by-step explanation:

So start off with 66 x 33. That will equal 2,178. Divide 2,178 by 18, like this: 2,178/18. That will equal 121. that will mean that 33 = 121. So 9 = 33 because 9 x 44 = 396 so you will divide that by 12 meaning that 9 equals 33. So now you will multiply 9 and 22 and then divide that answer by 33 making 6 = 22. now divide 22 by 6. That will equal 3.67. So 3.67 is the cost of one pie.

For the boxes above 12 and 44 it will be 16 = 59.

I hope it helped

please help me solve this i’ll mark brainliest

Answers

Answer:

Step-by-step explanation:

From the parallel lines, we know that corresponding angles are congruent, so:

(2y+78) = X

We also know that vertically opposite angles are equal:

X=5y

Substitute for X:

2y+78=5y

Solve:

2x-5x=-78

-3x=-78

x=26

Therefore, the measure of angle WXZ is 26.

bethany used data of the money she earned over the past 5 years to estimate the amount she would earn 10 years from now. this is an example of: group of answer choices

Answers

Extrapolation is a useful tool for predicting future values based on past data

Bethany used a method of predicting future values from past values known as extrapolation. Extrapolation is a powerful tool for predicting future values, but it is important to remember that it only works if the underlying trend in the data does not change. In Bethany's case, she is making an assumption that her income over the next 10 years will follow the same trend as it did in the past 5 years.

In extrapolation, the assumption is that the data points that were observed in the past will continue in the same direction and with the same intensity into the future. Therefore, extrapolation can be used to make predictions about future values even when no new data points have been collected. It is important to remember, however, that any predictions made using extrapolation are only estimates and may not be accurate.

Extrapolation is used in many different fields, such as finance, economics, medicine, engineering, and even weather forecasting. For example, stock analysts may use extrapolation to make predictions about future stock prices based on past prices. Similarly, meteorologists may use extrapolation to make predictions about future weather conditions based on historical weather data.

Overall, extrapolation is a useful tool for predicting future values based on past data. However, it is important to remember that it only works when the underlying trend in the data does not change. Additionally, any predictions made using extrapolation are only estimates and may not be accurate.

See more about extrapolation at: https://brainly.com/question/18768845

#SPJ11

Other Questions
10 ft20 ft15 ftFind the area.A = [?] ftRound to the nearesthundredth. all of these statements are true regarding bulimia nervosa and cognitive-behavioral therapy except that the therapist helps the patient to: Hydrogen (10 points)14. What does using hydrogen as a fuel source mean? How is it combined to create an energy source?15. Explain where hydrogen is found. What are sources of hydrogen found on Earth?16. Name and explain 3 advantages/disadvantages of using hydrogen as a fuel source and how it compares to using fossil fuels and other alternative energies (specifically include environmental issues that can happen.17. What is a fuel cell? How does it work? (include diagram)Solar Power (10 points)18. What is a good definition of solar energy?19. How does a solar cell make electricity? What is it made of?20. Name and explain 3 advantages/disadvantages about solar energy to heat or produce electricity compared to using fossil fuels or other alternatives (specifically include environmental issues that can happen.21. Find an example of solar energy being used in the U.S.? true or false: prior to the deadline, students should take this refund option into account when deciding to drop the course. when rotating the platform, the hanging mass should be removed from the platform. question 2 options: true false g suppose that you are an elementary school teacher and you are evaluating the reading levels of your students. you find an individual that reads 46.2 word per minute. you do some research and determine that the reading rates for their grade level are normally distributed with a mean of 85 words per minute and a standard deviation of 22 words per minute. a. at what percentile is the child's reading level (round final answer to one decimal place). What is cyanosis? Describe several physical and/or chemical factors that might create cyanosis in a patient and how cyanosis can be reversed. plea bargains a.are no longer a major part of the criminal justice system. b.have been openly discussed since the 1920s. c.were generally negotiated in secret late 1970s/early 1980s. d.were generally negotiated in secret in the 1920s. How is marine insurance different from automobile insurance? the volume of a cube (in cubic inches) plus three times the total length of its edges (in inches) is equal to twice its surface area (in square inches). how many inches long is its long diagonal? express your answer as where is a prime number. find . 1. How far will a car travel at 50 m/min in 25 min?2. How far will a car travel at 50 km/h in 7 h?3. How long will a ball roll 400 m at 70 m/min?4. If a car travels from 3 m/s to 30 m/s in 3 seconds, what is the car's average acceleration? what does studying artworks from the americas help us understand about the people that created them? T/F: Wolfram Alpha is an example of a search tool that can help you solve factual questions to help complete your homework. what is the time complexity of ukkonen's method for suffix tree construction, in terms of the size of text (n), having a constant-size alphabet ? the nurse is observing a child walk down stairs using a swing-through gait. what action by the child is correct? All Java applications must have a method __________.A. public static Main(String[] args)B. public static Main(String args[])C. public static void main(String[] args)D. public void main(String[] args)E. public static main(String[] args) Write a while loop that multiplies userValue by 2 while all of the following conditions are true: userValue is not 10 userValue is less than 25 Your Solution function userValue = AdjustValue(userValue) % Write a while loop that multiplies userValue by 2 % while userValue is not 10 and while userValue is less than 25 userValue = 0; end Run Your Solution Code to call your function when you click Run AdjustValue(2) the goal of the calculating an acceptance interval is to: a. find the probability that the true mean lies between two values. b. determine a range within which sample means are likely to occur, given a population mean and variance. c. determine a range within which no sample means would occur, given a population mean and variance. d. none of the above. ou and i each have a fair coin. i flip n times, and you flip n 1 times. you win if i get fewer heads than you. what is the probability that you win the game? In an aqueous solution of magnesium bromide, the solute is